LSAT and Law School Admissions Forum

Get expert LSAT preparation and law school admissions advice from PowerScore Test Preparation.

 Administrator
PowerScore Staff
  • PowerScore Staff
  • Posts: 8917
  • Joined: Feb 02, 2011
|
#72680
Complete Question Explanation

Flaw in the Reasoning, SN. The correct answer choice is (A).

Like all Flaw questions, here we are looking for an error in how the conclusion is drawn from the premises. As we are reading through the stimulus, it becomes clear that the premises here are using conditional reasoning, and that many of these conditional statements will "chain" together. (If you're struggling with identifying conditionals or "chaining" them together, please review the LSAT Bibles or Lesson 2 in the LSAT Course books). So we want to go ahead and express our premises in terms of conditionals:

Premise 1: democracy :arrow: free choces ("depends upon" is a necessary condition indicator)
Premise 2: free choices :arrow: well-reason opinions (remember that "unless" is a necessary condition indicator, but it requires the negation of the sufficient condition)
Premise 3: well-reasoned opinions :arrow: reading skills (in the Information Age)

A correct conclusion would chain all of these conditionals together in order to infer: democracy :arrow: reading skills (in the Information Age)

Or, in plain English it would be something like: in the Information Age, having a democracy requires reading skills. Instead, the conclusion in our stimulus has this backwards. The political scientist's conclusion is, in effect, saying: reading skills :arrow: democracy. Thus the conclusion in our stimulus has mistakenly reversed the conditional we arrived at by chaining together our premises (democracy :arrow: reading skills). A Mistaken Reversal like this can also known be referred to as "the confusion of sufficient and necessary conditions". (Again, if you want to review why the confusion of sufficient and necessary conditions is an error in reasoning, please review Lesson 2 in the LSAT Course Book).

The confusion of sufficient and necessary conditions is very frequently the correct answer for Flaw questions that involve conditional reasoning. Whenever you are working on a question like this, pay careful attention to see if the argument makes a Mistaken Reversal at any point. If it does (like the argument in our stimulus here does) - you'll want to Pre-Phrase (before even looking at the answer choices!) something like "confuses the sufficient and necessary conditions" as your answer. That way you can come down to the answer choices and easily select it. That is what we will do here.

Answer choice (A): This is the correct answer choice. This perfectly matches our Pre-Phrase and hits directly upon the error here: the political scientist's conclusion makes a Mistaken Reversal. It confuses the sufficient and necessary conditions of the conditional: democracy :arrow: reading skills (which we inferred by chaining together the premises).

Answer choice (B): Out of scope. This does not touch on the Flaw that we identified - the political scientist's confusion of sufficient and necessary. The argument is not flawed because it fails to take into account other means of forming well-reasoned opinions; the premises rule out any other means by telling us that reading is required for forming a well-reasoned opinion.

Answer choice (C): Out of scope. At no point does the argument mention anything about the reasons for doing something.

Answer choice (D): Out of scope. The argument at no point generalizes anything. It simply follows a series of conditional premises to make a Mistakenly Reversed conclusion.

Answer choice (E): This is the closest wrong answer, since it at least gets into the concept of sufficient and necessary conditions. But (E) is not describing the confusion of sufficient and necessary conditions, which is the Flaw in our stimulus. In fact, what (E) is describing isn't even a Flaw; the "Flaw" in (E) is actually sound logic. Here is a longer way of describing what (E) is saying: "A is sufficient to bring about B. C and D are necessary for B to occur. It would be a Flaw to say that A is sufficient to bring about C and D." But in that case it would actually be correct to say that A is sufficient to bring about C and D. Why? If A :arrow: B and B :arrow: C and D, then we can chain those conditionals together to say A :arrow: C and D. To put it in terms of the language involved in (E): a condition under which something occurs must logically also be a condition under which its prerequisites occur.

Hope that helps!
 mallie
  • Posts: 7
  • Joined: Nov 28, 2019
|
#74908
Could someone explain the functional difference(s) between A and E? I am having a hard time understanding what they mean and why A is better than E.

Thanks in advance!
 Paul Marsh
PowerScore Staff
  • PowerScore Staff
  • Posts: 290
  • Joined: Oct 15, 2019
|
#74947
Hi mallie! I went ahead and posted a full explanation for this problem above. Let me know if you have any further questions about this one. Hope that helps!

Get the most out of your LSAT Prep Plus subscription.

Analyze and track your performance with our Testing and Analytics Package.